Liegt Weinberg mit dieser Darstellung falsch, wie Machs Prinzip in die Allgemeine Relativitätstheorie einfließt?

In seinem Buch "Dreams of a Final Theory" S.144 sagt Steven Weinberg

„Die Zirkulation der Materie des Universums um den Zenit, die von Beobachtern auf einem Karussell gesehen wird, erzeugt ein Feld, das dem Magnetfeld ähnelt, das durch die Zirkulation von Elektrizität in den Spulen eines Elektromagneten erzeugt wird. Es ist dieses gravitomagnetische Feld, das in Der Karussell-Bezugsrahmen erzeugt die Effekte, die in konventionelleren Bezugsrahmen der Zentrifugalkraft zugeschrieben werden.

Er fährt fort, dass wir deshalb in der Allgemeinen Relativitätstheorie nicht unterscheiden können, ob wir uns in einem Rotationslabor befinden oder nicht.

Dies scheint eine kühne und falsche Akzeptanz einer sehr starken Version von Machs Prinzip zu sein

"Alle Zentrifugaleffekte werden durch die Gravitationseffekte der rotierenden Sterne verursacht, wenn sie vom rotierenden Referenzrahmen aus gesehen werden."

Weinbergs Bericht scheint modernen Ansichten über Machs Prinzip zu widersprechen. Ja, es gibt den Lense-Thirring-Effekt und Frame Dragging, aber das sagt noch lange nicht aus, dass alle Trägheitseffekte in einem rotierenden Bezugssystem durch die Bewegung der umgebenden Materie im Universum verursacht werden.

Können wir nicht feststellen, ob wir uns in einem sich drehenden Raumschiff befinden oder nicht (unter Verwendung des Zentrifugal- und Trägheitseffekts), ohne Bezug auf die fernen Sterne? Würde Weinbergs Behauptung nicht bedeuten, dass, wenn wir die Masse des umgebenden Universums gegen Null gehen lassen, der Zentrifugaleffekt im rotierenden Laborrahmen ebenfalls gegen Null gehen würde?

Kurz gesagt, liegt Weinberg hier in Bezug auf Machs Prinzip in der allgemeinen Relativitätstheorie falsch?

Haben Sie diese Ansicht von „lang toten Ideen“ gelesen physical.stackexchange.com/questions/5483/… .
Ich habe gerade Ihren Punkt angesprochen, indem ich eine alte Frage beantwortet habe: physical.stackexchange.com/a/362242/154997
@ Countto10 Ja, ich habe es gelesen und es scheint zu sagen, dass Weinberg falsch liegt. Ich hätte gedacht, Weinberg wüsste es besser. Er hat ein sehr berühmtes Buch über GR geschrieben. Es fällt mir schwer zu glauben, dass er so falsch liegt, also stelle ich mein eigenes Verständnis in Frage.
Was, wenn es keine Sterne im Universum gibt?
Weinbergs Buch ist sehr alt, mein einziger Gedanke ist, dass er angesichts des Titels versuchte, inklusiv zu sein, und möglicherweise versuchte er angesichts seines Rufs, die Diskussion über ein Thema, für das er eine Vorliebe hatte, wieder in Gang zu bringen. Wenn er es seitdem nicht abgedeckt hat, ist er wohl weitergezogen.
Ohne weiteren Kontext ist es sehr schwierig zu beurteilen, ob Weinberg richtig oder falsch liegt. Aus dem Material in der Frage scheinen Sie derjenige zu sein, der seine Ideen als Bestätigung einer starken Version von Machs Prinzip interpretiert.
Ich bin normalerweise kein Wettmann, aber in diesem Fall bin ich bereit zu wetten, dass Weinberg hier keinen Fehler macht.

Antworten (1)

Brans-Dickes Originalarbeit über ihre Skalar-Tensor-Version von GR beginnt mit einer Untersuchung dieser Frage, und ich empfehle dringend, sie zu lesen – es ist ein wunderbares Beispiel dafür, wie man eine wissenschaftliche Arbeit schreibt, die fast jeder verstehen kann

In meiner Lektüre deuten sie im Grunde darauf hin, dass Mach falsch liegt, zumindest in Bezug auf die Quelle des "Feldes". Sie geben das Beispiel von jemandem in einem Würfel, der sich aus dem Fenster lehnt und ein Gewehr tangential zum Würfel schießt. Dadurch dreht sich der Würfel und sollte daher Zentripetalkräfte erzeugen. Laut Mach ist die Quelle dafür jedoch der "Hintergrund", der in diesem Fall eine einzelne 22-Kugel ist. Wie kommt es, dass dies einen solchen Effekt haben kann, verglichen mit, sagen wir, der Masse der Wände des Würfels?

Das Problem existiert also, wenn Sie es so sehen, aber sie schlagen vor, dass Machs Lösung falsch ist.

Ich habe mir nur kurz das Papier von Barns and Dicke angesehen. Nach Ihrer Erklärung denke ich jedoch, dass es im Barns-Dicke-Beispiel ein grundlegendes Missverständnis über Machs Prinzip geben könnte. Nach Machs Prinzip dürfen Sie sich kein 22-Kaliber-Gewehr oder irgendein Labor mit massiven Wänden in einem leeren Raum (im Hintergrund) vorstellen, denn wenn Sie laut Mach alle Massen aus dem Universum eliminieren, mit Ausnahme einiger weniger Objekte (Labor & Gewehr & Kugel) würden diese Objekte masselos werden , da die Trägheitsmasse von der globalen Verteilung der Materie beeinflusst wird.
Daher scheint sich das Gyroskop entgegen der Behauptung von Barns & Dicke nicht relativ zu den Wänden des Labors zu drehen (Linear- und Drehimpuls sind alle Null). Ich denke, dass Machs Prinzip implizit besagt, dass es nicht möglich ist, traditionelle physikalische Gesetze in einem leeren Raum (Universum) anzuwenden.
Schöner Artikelhinweis! Danke, es ist ein Juwel.